Sufficient & Necessary Questions - - Question 7
If the city council maintains spending at the same level as this year's, it can be expected to levy a sales tax of 2 ...
Replies
Mehran October 8, 2013
Thanks for your question. The video explanation for this question is now live inside of LSATMax.Hope this helps! Please let us know if you have any other questions.
180 October 9, 2013
Thank you for the fast responseChulitazo August 5, 2014
Although I agree that answer choice C is the correct answer, I disagree with the video explanation that the original deduction is not necessarily true whereas answer choice C is absolutely certain. Below I offer my deductions:Original Deduction:
PR: S constant - > 2% ST
C: not 2% ST - > S increases
This conclusion is not necessarily true, and assumes that spending would increase instead of remaining constant. Nevertheless, it is true that spending did not remain constant.
Answer Choice C Deduction:
PR: not WW increase - > P constant
C: P increases - > WW increases
Again, this is not necessarily true and assumes that prices will increase instead of remaining constant. However, in any case, it is true that prices did not remain constant.
Would you please explain to me my error in reasoning here? Thank you in advance.
Naz August 12, 2014
So, as discussed in the video, the main issue with the correct answer is that its condition that is a "constant" is in the necessary condition position, whereas the "constant" condition in the stimulus is in the sufficient condition position. Let's discuss:The stimulus tells us that if the city council's spending stays constant, then sales taxes can be expected to be levied by 2% next year.
PR: SC ==> 2%I
We are then told that if the council does not levy taxes by 2%, then it will be because the council is increasing its spending.
C: not 2%I ==> SI
So what is the contrapositive of the principle rule in the stimulus?
"Sales tax not levied by 2% next year, then spending on council is not constant."
Contrapositive: not 2%I ==> not SC
As you can see, the conclusion is saying that if "not 2%I" then spending will increase. Spending not being constant does not necessarily have to be spending increasing. It could be spending decreasing. This is why the premise is not necessarily true, since it is not exactly a correct contrapositive.
Now let's look at answer choice (C).
We know that if companies do not increase workers' wages, then the prices they charge will remain constant.
PR (C): not WWI ==> PC
The conclusion states that if the companies prices do increase (i.e. they are not constant), it will be because they have increased wages.
C (C): not PC ==> WWI
The conclusion in answer choice (C) is the contrapositive of the principle rule. You may be wondering why we write it out: "not PC" as opposed to "PI." Well, prices increasing guarantees that the price is not constant. However, we could not do the same in the stimulus because spending not being constant could be spending increasing or decreasing.
Though answer choice (C) and the stimulus are not exactly the same (due to the constant being switched), it is still the answer choice that has the reasoning closest to the reasoning in the stimulus, i.e. it is our correct answer.
Hope that clears things up! Please let us know if you have any other questions.